Suppose $x, y, z$ are positive real number such that $x + 2y + 3z = 1$. Find the maximum value of $xyz^2$












1












$begingroup$


I'm trying to solve this problem using Lagrange's multiplier method but I'm unable to get the value of lambda and also $z.$ kindly help me out with this problem.
I'm getting $x = 2y$



I'm stuck somewhere between the steps










share|cite|improve this question











$endgroup$

















    1












    $begingroup$


    I'm trying to solve this problem using Lagrange's multiplier method but I'm unable to get the value of lambda and also $z.$ kindly help me out with this problem.
    I'm getting $x = 2y$



    I'm stuck somewhere between the steps










    share|cite|improve this question











    $endgroup$















      1












      1








      1


      1



      $begingroup$


      I'm trying to solve this problem using Lagrange's multiplier method but I'm unable to get the value of lambda and also $z.$ kindly help me out with this problem.
      I'm getting $x = 2y$



      I'm stuck somewhere between the steps










      share|cite|improve this question











      $endgroup$




      I'm trying to solve this problem using Lagrange's multiplier method but I'm unable to get the value of lambda and also $z.$ kindly help me out with this problem.
      I'm getting $x = 2y$



      I'm stuck somewhere between the steps







      maxima-minima






      share|cite|improve this question















      share|cite|improve this question













      share|cite|improve this question




      share|cite|improve this question








      edited May 1 '18 at 6:52









      User

      1,88221229




      1,88221229










      asked May 1 '18 at 5:44









      Priya WadhwaPriya Wadhwa

      789




      789






















          4 Answers
          4






          active

          oldest

          votes


















          2












          $begingroup$

          By definition, $lambda$ is a constant, and we don't even really care what $lambda$ is. It is always linear and factored from the gradient, so instead of finding $lambda$, use it to jump from one equation to another.



          $lambda=yz^2$



          $lambda=frac{xz^2}{2}$



          $lambda=frac{2xyz}{3}$






          share|cite|improve this answer









          $endgroup$









          • 1




            $begingroup$
            by equating values of lambda and the constraint x + 2y + 3z = 1 the values of x,y,z can be calculated and i got the answer x = 1/4, y= 1/8 and z = 1/6.
            $endgroup$
            – Priya Wadhwa
            May 1 '18 at 6:41



















          2












          $begingroup$

          Alt. hint (no calculus):   the following inequality holds by AM-GM, with equality iff $displaystyle x = 2y = frac{3}{2}z,$:



          $$
          frac{sqrt{3}}{sqrt[4]{2}} cdot sqrt[4]{xyz^2} = sqrt[4]{x cdot 2y cdot frac{3}{2}z cdot frac{3}{2}z} ;le; frac{x+2y+3z}{4} = frac{1}{4} ;;iff;;xyz^2 le frac{2}{3^2 cdot 4^4} = frac{1}{1152}
          $$






          share|cite|improve this answer









          $endgroup$





















            1












            $begingroup$

            Hint: prove that $$xyz^2le frac{1}{1152}$$ and the equal sign holds if $$x=frac{1}{4},y=frac{1}{8},z=frac{1}{6}$$






            share|cite|improve this answer









            $endgroup$





















              1












              $begingroup$

              You can also convert $xyz^2$ into a function of two variables $f(y,z)=(1-2y-3z)yz^2$ and use this test to conclude that the maxima occurs at $(frac{1}{4},frac{1}{8}, frac{1}{6})$ and is $1152.$ You can ignore the other critical points since all of them have at least one $0$ entry while $x,y$ and $z$ are given positive.






              share|cite|improve this answer









              $endgroup$














                Your Answer





                StackExchange.ifUsing("editor", function () {
                return StackExchange.using("mathjaxEditing", function () {
                StackExchange.MarkdownEditor.creationCallbacks.add(function (editor, postfix) {
                StackExchange.mathjaxEditing.prepareWmdForMathJax(editor, postfix, [["$", "$"], ["\\(","\\)"]]);
                });
                });
                }, "mathjax-editing");

                StackExchange.ready(function() {
                var channelOptions = {
                tags: "".split(" "),
                id: "69"
                };
                initTagRenderer("".split(" "), "".split(" "), channelOptions);

                StackExchange.using("externalEditor", function() {
                // Have to fire editor after snippets, if snippets enabled
                if (StackExchange.settings.snippets.snippetsEnabled) {
                StackExchange.using("snippets", function() {
                createEditor();
                });
                }
                else {
                createEditor();
                }
                });

                function createEditor() {
                StackExchange.prepareEditor({
                heartbeatType: 'answer',
                autoActivateHeartbeat: false,
                convertImagesToLinks: true,
                noModals: true,
                showLowRepImageUploadWarning: true,
                reputationToPostImages: 10,
                bindNavPrevention: true,
                postfix: "",
                imageUploader: {
                brandingHtml: "Powered by u003ca class="icon-imgur-white" href="https://imgur.com/"u003eu003c/au003e",
                contentPolicyHtml: "User contributions licensed under u003ca href="https://creativecommons.org/licenses/by-sa/3.0/"u003ecc by-sa 3.0 with attribution requiredu003c/au003e u003ca href="https://stackoverflow.com/legal/content-policy"u003e(content policy)u003c/au003e",
                allowUrls: true
                },
                noCode: true, onDemand: true,
                discardSelector: ".discard-answer"
                ,immediatelyShowMarkdownHelp:true
                });


                }
                });














                draft saved

                draft discarded


















                StackExchange.ready(
                function () {
                StackExchange.openid.initPostLogin('.new-post-login', 'https%3a%2f%2fmath.stackexchange.com%2fquestions%2f2761322%2fsuppose-x-y-z-are-positive-real-number-such-that-x-2y-3z-1-find-the%23new-answer', 'question_page');
                }
                );

                Post as a guest















                Required, but never shown

























                4 Answers
                4






                active

                oldest

                votes








                4 Answers
                4






                active

                oldest

                votes









                active

                oldest

                votes






                active

                oldest

                votes









                2












                $begingroup$

                By definition, $lambda$ is a constant, and we don't even really care what $lambda$ is. It is always linear and factored from the gradient, so instead of finding $lambda$, use it to jump from one equation to another.



                $lambda=yz^2$



                $lambda=frac{xz^2}{2}$



                $lambda=frac{2xyz}{3}$






                share|cite|improve this answer









                $endgroup$









                • 1




                  $begingroup$
                  by equating values of lambda and the constraint x + 2y + 3z = 1 the values of x,y,z can be calculated and i got the answer x = 1/4, y= 1/8 and z = 1/6.
                  $endgroup$
                  – Priya Wadhwa
                  May 1 '18 at 6:41
















                2












                $begingroup$

                By definition, $lambda$ is a constant, and we don't even really care what $lambda$ is. It is always linear and factored from the gradient, so instead of finding $lambda$, use it to jump from one equation to another.



                $lambda=yz^2$



                $lambda=frac{xz^2}{2}$



                $lambda=frac{2xyz}{3}$






                share|cite|improve this answer









                $endgroup$









                • 1




                  $begingroup$
                  by equating values of lambda and the constraint x + 2y + 3z = 1 the values of x,y,z can be calculated and i got the answer x = 1/4, y= 1/8 and z = 1/6.
                  $endgroup$
                  – Priya Wadhwa
                  May 1 '18 at 6:41














                2












                2








                2





                $begingroup$

                By definition, $lambda$ is a constant, and we don't even really care what $lambda$ is. It is always linear and factored from the gradient, so instead of finding $lambda$, use it to jump from one equation to another.



                $lambda=yz^2$



                $lambda=frac{xz^2}{2}$



                $lambda=frac{2xyz}{3}$






                share|cite|improve this answer









                $endgroup$



                By definition, $lambda$ is a constant, and we don't even really care what $lambda$ is. It is always linear and factored from the gradient, so instead of finding $lambda$, use it to jump from one equation to another.



                $lambda=yz^2$



                $lambda=frac{xz^2}{2}$



                $lambda=frac{2xyz}{3}$







                share|cite|improve this answer












                share|cite|improve this answer



                share|cite|improve this answer










                answered May 1 '18 at 6:14









                AEngineerAEngineer

                1,6521318




                1,6521318








                • 1




                  $begingroup$
                  by equating values of lambda and the constraint x + 2y + 3z = 1 the values of x,y,z can be calculated and i got the answer x = 1/4, y= 1/8 and z = 1/6.
                  $endgroup$
                  – Priya Wadhwa
                  May 1 '18 at 6:41














                • 1




                  $begingroup$
                  by equating values of lambda and the constraint x + 2y + 3z = 1 the values of x,y,z can be calculated and i got the answer x = 1/4, y= 1/8 and z = 1/6.
                  $endgroup$
                  – Priya Wadhwa
                  May 1 '18 at 6:41








                1




                1




                $begingroup$
                by equating values of lambda and the constraint x + 2y + 3z = 1 the values of x,y,z can be calculated and i got the answer x = 1/4, y= 1/8 and z = 1/6.
                $endgroup$
                – Priya Wadhwa
                May 1 '18 at 6:41




                $begingroup$
                by equating values of lambda and the constraint x + 2y + 3z = 1 the values of x,y,z can be calculated and i got the answer x = 1/4, y= 1/8 and z = 1/6.
                $endgroup$
                – Priya Wadhwa
                May 1 '18 at 6:41











                2












                $begingroup$

                Alt. hint (no calculus):   the following inequality holds by AM-GM, with equality iff $displaystyle x = 2y = frac{3}{2}z,$:



                $$
                frac{sqrt{3}}{sqrt[4]{2}} cdot sqrt[4]{xyz^2} = sqrt[4]{x cdot 2y cdot frac{3}{2}z cdot frac{3}{2}z} ;le; frac{x+2y+3z}{4} = frac{1}{4} ;;iff;;xyz^2 le frac{2}{3^2 cdot 4^4} = frac{1}{1152}
                $$






                share|cite|improve this answer









                $endgroup$


















                  2












                  $begingroup$

                  Alt. hint (no calculus):   the following inequality holds by AM-GM, with equality iff $displaystyle x = 2y = frac{3}{2}z,$:



                  $$
                  frac{sqrt{3}}{sqrt[4]{2}} cdot sqrt[4]{xyz^2} = sqrt[4]{x cdot 2y cdot frac{3}{2}z cdot frac{3}{2}z} ;le; frac{x+2y+3z}{4} = frac{1}{4} ;;iff;;xyz^2 le frac{2}{3^2 cdot 4^4} = frac{1}{1152}
                  $$






                  share|cite|improve this answer









                  $endgroup$
















                    2












                    2








                    2





                    $begingroup$

                    Alt. hint (no calculus):   the following inequality holds by AM-GM, with equality iff $displaystyle x = 2y = frac{3}{2}z,$:



                    $$
                    frac{sqrt{3}}{sqrt[4]{2}} cdot sqrt[4]{xyz^2} = sqrt[4]{x cdot 2y cdot frac{3}{2}z cdot frac{3}{2}z} ;le; frac{x+2y+3z}{4} = frac{1}{4} ;;iff;;xyz^2 le frac{2}{3^2 cdot 4^4} = frac{1}{1152}
                    $$






                    share|cite|improve this answer









                    $endgroup$



                    Alt. hint (no calculus):   the following inequality holds by AM-GM, with equality iff $displaystyle x = 2y = frac{3}{2}z,$:



                    $$
                    frac{sqrt{3}}{sqrt[4]{2}} cdot sqrt[4]{xyz^2} = sqrt[4]{x cdot 2y cdot frac{3}{2}z cdot frac{3}{2}z} ;le; frac{x+2y+3z}{4} = frac{1}{4} ;;iff;;xyz^2 le frac{2}{3^2 cdot 4^4} = frac{1}{1152}
                    $$







                    share|cite|improve this answer












                    share|cite|improve this answer



                    share|cite|improve this answer










                    answered May 1 '18 at 7:05









                    dxivdxiv

                    58k648102




                    58k648102























                        1












                        $begingroup$

                        Hint: prove that $$xyz^2le frac{1}{1152}$$ and the equal sign holds if $$x=frac{1}{4},y=frac{1}{8},z=frac{1}{6}$$






                        share|cite|improve this answer









                        $endgroup$


















                          1












                          $begingroup$

                          Hint: prove that $$xyz^2le frac{1}{1152}$$ and the equal sign holds if $$x=frac{1}{4},y=frac{1}{8},z=frac{1}{6}$$






                          share|cite|improve this answer









                          $endgroup$
















                            1












                            1








                            1





                            $begingroup$

                            Hint: prove that $$xyz^2le frac{1}{1152}$$ and the equal sign holds if $$x=frac{1}{4},y=frac{1}{8},z=frac{1}{6}$$






                            share|cite|improve this answer









                            $endgroup$



                            Hint: prove that $$xyz^2le frac{1}{1152}$$ and the equal sign holds if $$x=frac{1}{4},y=frac{1}{8},z=frac{1}{6}$$







                            share|cite|improve this answer












                            share|cite|improve this answer



                            share|cite|improve this answer










                            answered May 1 '18 at 5:53









                            Dr. Sonnhard GraubnerDr. Sonnhard Graubner

                            78.8k42867




                            78.8k42867























                                1












                                $begingroup$

                                You can also convert $xyz^2$ into a function of two variables $f(y,z)=(1-2y-3z)yz^2$ and use this test to conclude that the maxima occurs at $(frac{1}{4},frac{1}{8}, frac{1}{6})$ and is $1152.$ You can ignore the other critical points since all of them have at least one $0$ entry while $x,y$ and $z$ are given positive.






                                share|cite|improve this answer









                                $endgroup$


















                                  1












                                  $begingroup$

                                  You can also convert $xyz^2$ into a function of two variables $f(y,z)=(1-2y-3z)yz^2$ and use this test to conclude that the maxima occurs at $(frac{1}{4},frac{1}{8}, frac{1}{6})$ and is $1152.$ You can ignore the other critical points since all of them have at least one $0$ entry while $x,y$ and $z$ are given positive.






                                  share|cite|improve this answer









                                  $endgroup$
















                                    1












                                    1








                                    1





                                    $begingroup$

                                    You can also convert $xyz^2$ into a function of two variables $f(y,z)=(1-2y-3z)yz^2$ and use this test to conclude that the maxima occurs at $(frac{1}{4},frac{1}{8}, frac{1}{6})$ and is $1152.$ You can ignore the other critical points since all of them have at least one $0$ entry while $x,y$ and $z$ are given positive.






                                    share|cite|improve this answer









                                    $endgroup$



                                    You can also convert $xyz^2$ into a function of two variables $f(y,z)=(1-2y-3z)yz^2$ and use this test to conclude that the maxima occurs at $(frac{1}{4},frac{1}{8}, frac{1}{6})$ and is $1152.$ You can ignore the other critical points since all of them have at least one $0$ entry while $x,y$ and $z$ are given positive.







                                    share|cite|improve this answer












                                    share|cite|improve this answer



                                    share|cite|improve this answer










                                    answered Feb 1 at 4:10









                                    RhaldrynRhaldryn

                                    352416




                                    352416






























                                        draft saved

                                        draft discarded




















































                                        Thanks for contributing an answer to Mathematics Stack Exchange!


                                        • Please be sure to answer the question. Provide details and share your research!

                                        But avoid



                                        • Asking for help, clarification, or responding to other answers.

                                        • Making statements based on opinion; back them up with references or personal experience.


                                        Use MathJax to format equations. MathJax reference.


                                        To learn more, see our tips on writing great answers.




                                        draft saved


                                        draft discarded














                                        StackExchange.ready(
                                        function () {
                                        StackExchange.openid.initPostLogin('.new-post-login', 'https%3a%2f%2fmath.stackexchange.com%2fquestions%2f2761322%2fsuppose-x-y-z-are-positive-real-number-such-that-x-2y-3z-1-find-the%23new-answer', 'question_page');
                                        }
                                        );

                                        Post as a guest















                                        Required, but never shown





















































                                        Required, but never shown














                                        Required, but never shown












                                        Required, but never shown







                                        Required, but never shown

































                                        Required, but never shown














                                        Required, but never shown












                                        Required, but never shown







                                        Required, but never shown







                                        Popular posts from this blog

                                        Can a sorcerer learn a 5th-level spell early by creating spell slots using the Font of Magic feature?

                                        Does disintegrating a polymorphed enemy still kill it after the 2018 errata?

                                        A Topological Invariant for $pi_3(U(n))$